User contributions for Wescarroll
Results for Wescarroll talk block log uploads logs
A user with 34 edits. Account created on 17 February 2021.
23 February 2024
- 19:2919:29, 23 February 2024 diff hist −2 m 2023 AMC 8 Problems/Problem 15 typographical fix
- 19:2819:28, 23 February 2024 diff hist +22 m 2017 AMC 10A Problems/Problem 22 →Note
- 19:2519:25, 23 February 2024 diff hist +21 m 2011 AMC 12A Problems/Problem 8 →Podcast Solution
8 February 2024
- 18:4118:41, 8 February 2024 diff hist +11 m 2024 AIME I Problems/Problem 13 Added link to Fermat's Little Theorem
- 18:3918:39, 8 February 2024 diff hist +29 m 2024 AIME I Problems/Problem 13 "Easy," my eyeteeth
7 February 2024
- 18:4618:46, 7 February 2024 diff hist +16 2024 AMC 8 Problems/Problem 24 Correcting labelling error
6 February 2024
- 20:0320:03, 6 February 2024 diff hist +143 2024 AIME I Problems/Problem 10 They aren't asking for AP; they are asking for m+n. So I copied the original problem statement into the copy of the problem statement on the solution page.
21 January 2024
- 16:1016:10, 21 January 2024 diff hist +4 m 2023 AMC 8 Problems →Problem 20
- 15:4815:48, 21 January 2024 diff hist +207 m 2023 AMC 8 Problems/Problem 15 →Solution 2
18 January 2024
- 17:5917:59, 18 January 2024 diff hist +29 Lifting the Exponent No edit summary
17 January 2024
- 18:5618:56, 17 January 2024 diff hist +9 2017 AMC 10A Problems/Problem 22 →Note
- 18:5518:55, 17 January 2024 diff hist +83 2017 AMC 10A Problems/Problem 22 →Note
- 18:2918:29, 17 January 2024 diff hist +14 2017 AMC 10A Problems/Problem 22 →Note
- 18:2818:28, 17 January 2024 diff hist 0 2017 AMC 10A Problems/Problem 22 →Note
- 18:2718:27, 17 January 2024 diff hist +94 2017 AMC 10A Problems/Problem 22 →Note
- 18:2618:26, 17 January 2024 diff hist +241 2017 AMC 10A Problems/Problem 22 →Note
11 January 2024
- 18:2218:22, 11 January 2024 diff hist +40 m Lifting the Exponent No edit summary
- 18:2118:21, 11 January 2024 diff hist +437 N Lifting the Exponent Created page with "Let <math>p</math> be an odd prime, and let <math>a</math> and <math>b</math> be integers relatively prime to <math>p</math> such that <math>p \mid (a-b)</math>. Let <math>n</..."
26 September 2022
- 15:4815:48, 26 September 2022 diff hist +162 2007 AMC 12A Problems/Problem 16 No edit summary
- 15:4015:40, 26 September 2022 diff hist −8 2011 AMC 12A Problems/Problem 8 No edit summary
- 15:3915:39, 26 September 2022 diff hist +58 2011 AMC 12A Problems/Problem 8 No edit summary
- 15:3815:38, 26 September 2022 diff hist +85 2011 AMC 12A Problems/Problem 8 No edit summary